Please answer!


What's the inverse of f(x) = 3x^3 - 4?

Please Answer! What's The Inverse Of F(x) = 3x^3 - 4?

Answers

Answer 1

Answer:

C i know this for a fact

Step-by-step explanation:

hope this helps ''whole lotta love'' NUNU

Answer 2

Answer:

answer is c

Step-by-step explanation:

took the test lol


Related Questions

Somebody please help me. I cant figure this out once so ever.

Answers

Answer:

A

Step-by-step explanation:

The Fibonacci sequence is the sum of the two terms before it. It looks like this:

1, 1, 2, 3, 5, 8, 13, 21, 34, ...

As you can see, 13 and 21 are indeed consecutive terms.

lcm of 6 and 13 what is it?

Answers

Answer:

78

Step-by-step explanation:

Since 13 is a prime, the least common multiple of it and 6 is their product. 6*13=78. Hope this helps!

Answer:

78

Step-by-step explanation:

Least Common Multiple of 6 and 13. Least common multiple (LCM) of 6 and 13 is 78.

6- 6, 12, 18, 24, 30, 36, 42, 48, 54, 60, 66, 72, 78

13- 13, 26, 39, 52, 65, 78

Which of these is equal to 0.13?
13
A.
99
12
B.
90
c. 1

Answers

Answer:

Is this the inter question

Which two numbers doesStartRoot 61 EndRoot lie between on a number line? 7.7 and 7.8 7.8 and 7.9 7.9 and 8.0 8.0 and 8.1

Answers

Answer:

Answer is 7.8 and 7.9

Step-by-step explanation:

Answer:

the answer is B 7.8 to 7.9 have fun

Step-by-step explanation:

What has to be added to the sum of 4/7 and 7/4 to get 3?

Answers

Answer:

if you add 1 u get 3

Step-by-step explanation

Answer:

[tex]0.67858[/tex]

Step-by-step explanation:

[tex]\frac{4}{7}+\frac{7}{4}\\\mathrm{For}\:\frac{4}{7}:\:\mathrm{multiply\:the\:denominator\:and\:numerator\:by\:}\:4\\\frac{4}{7}=\frac{4\cdot \:4}{7\cdot \:4}=\frac{16}{28}\\\mathrm{For}\:\frac{7}{4}:\:\mathrm{multiply\:the\:denominator\:and\:numerator\:by\:}\:7\\\frac{7}{4}=\frac{7\cdot \:7}{4\cdot \:7}=\frac{49}{28}\\=\frac{16}{28}+\frac{49}{28}\\\mathrm{Since\:the\:denominators\:are\:equal,\:combine\:the\:fractions}:\quad \frac{a}{c}\pm \frac{b}{c}=\frac{a\pm \:b}{c}\\=\frac{16+49}{28}[/tex]

[tex]\mathrm{Add\:the\:numbers:}\:16+49=65\\=\frac{65}{28}\\\mathrm{Decimal:\quad }\:2.32142\\3-2.32142=0.67858[/tex]

PLEASE HELP!! The question is “ Abbey spends 5 hours less then triple the amount of time Emily spends on homework. Nathan spends twice as much time on homework as Abbey does. Write a simplified expression for the total number of hours the three friends spend on their homework. Show your work.”

Answers

Answer:

5(2x-3)

Step-by-step explanation:

Let the amount of time Emily spends on homework.=x (in hours)

Abbey's Time = 5 hours less than triple the amount of time Emily spends

Therefore:

Amount of Time spent by Abbey =3x-5

Nathan's Time = Twice as much time as Abbey.

Amount of Time spent by Nathan =2(3x-5)

Next, we calculate the total number of hours the three friends spend on their homework

=Time spent by Emily+Time spent by Abbey+Time spent by Nathan

=x+(3x-5)+2(3x-5)

=x+3x-5+6x-10

Collect like terms

=x+3x+6x-5-10

=10x-15

Next, we simplify by factorizing

Therefore, Total number of hours =5(2x-3)

Find the value of x in the question

Answers

Answer:

x=52

Step-by-step explanation:

r=90 s=34  q=180-(90+34)

180-(90+34)=56

to find angle sqr

180-56=124

124=x+72

52=x

Answer:

x = 52

Step-by-step explanation:

→We are given an exterior angle (outside of the triangle) as (x + 72). We are also given two interior angles as 34 and 90. The sum of the two farthest interior angles away from the exterior, is equal to the exterior. So this means that ∠R(90°) + ∠S(34°) = ∠PQS(x + 72), and we can set up the equation like that:

90 + 34 = x + 72

→Add like terms (90 and 34):

124 = x + 72

→Subtract 72 from both sides:

52 = x

What is the value of this expression when c= -4 and d = 10?
(c+d2)
O A.
2.
OB.
9
Ос.
21
OD. 41

Answers

Answer:

[tex]c + d^2 = 96[/tex]

Step-by-step explanation:

Given

[tex]c = -4\\d = 10[/tex]

Required

Find [tex]c + d^2[/tex]

To find [tex]c + d^2[/tex], we simply substitute 10 for d and -4 for c in the given expression.

The step is as follows:

[tex]c + d^2[/tex]

Substitute [tex]c = -4\\d = 10[/tex]

[tex]c + d^2 = -4 + 10^2[/tex]

[tex]c + d^2 = -4 + 10*10[/tex]

[tex]c + d^2 = -4 + 100[/tex]

[tex]c + d^2 = 96[/tex]

The expression cannot be solved any further; so, we can conclude that when d = 10 and c = -4, [tex]c + d^2 = 96[/tex]

For which set of data will the scatter plot represent a positive linear association between x and y?
O Set A
O Set B
O Setc
O Set D
PLEASE HELP ME

Answers

Answer:

Set C

Step-by-step explanation:

Answer:

The answer is Set B

Step-by-step explanation:  

Positive linear association goes from left to right and in a straight line. If you put the information into a scatter plot, the line moves in the positive direction and is in a straight line. (In a scatter plot, x goes across the bottom and y is vertical.)

Choose the point on the terminal side of -45°.

a. (-3, -3)
b. (4, -4)
c. (5, 5)
d. (-2, 2)

Answers

Answer:

B

Step-by-step explanation:

if you translate the cubic parent function, F(x) down 2 units, what is the equation of the new parent function

Answers

Answer:

y=f(x)-2

Step-by-step explanation:

sarah wants to open up her own coffee shop. It costs her $3 to make each cup of coffee, plus she has to pay a $24 renter's fee for her shop. She sells the coffee for $5 per cup. How many cups of coffee does she has to sell before she can start making a profit? *

Answers

Answer:

5

Step-by-step explanation:

So you divide 24 by 5 and you get 4.8 so you round it up to 5

50 POINTS IF YOU GET THIS QUESTION!!! Find area of the following shape if the vertices are X=(1,1), Y=(2,5), Z=(5,5), W=(8,1)

Answers

Answer:

20

Step-by-step explanation:

To find area using coordinates, you would take the y-value of the first coordinate and the x-value of the second coordinate and multiply them. Then, take the x-value of the first coordinate and multiply it with the second y-coordinate. Take the second number and subtract it from the first number. for multiple coordinates, you would multiply the first and the second, the second and the third, and so on. Once you reach the last coordinate, just multiply it by the first again. then, take all of the numbers, from multiplying your coordinates and add them together and divide them by 2 to find your area.

We would start with coordinate (8,1) here because it's letter variable is W, which is first of the others alphabetically.

(8,1) (1,1)

1 x 1 = 1

8 x 1 = 8

1 - 8 = -7

next coordinate:

(1,1) (2,5)

1 x 2 = 2

1 x 5 = 5

2 - 5 = -3

next:

(2,5) (5,5)

5 x 5 = 25

2 x 5 = 10

25 - 10 = 15

next:

(5,5) (8,1)

5 x 5 = 40

5 x 1 = 5

40 - 5 = 35

-7 - 3 + 15 + 35 = 40

40/2 = 20

Answer:

[tex]\boxed{\mathrm{view \: attachment}}[/tex]

Step-by-step explanation:

Let M(x)=4x2−3x and N(x)=−5x3−6x2−3. Find M(x)−N(x)

Answers

Answer:

5x³+10x²-3x+3

Step-by-step explanation:

(4x²-3x) - (-5x³-6x²-3)

5x³+10x²-3x+3

If equations M(x)=[tex]4x^{2} -3x[/tex] and N(x)=[tex]-5x^{3}-6x^{2} -3[/tex] then M(x)-N(x)=[tex]5x^{3} +10x^{2} -3x+3[/tex].

What is equation?

An equation is a relationship between two or more variables expressed in equal to form. It is often equated to find the values of variables. The form of an equation is ax+by=c.

How solve equations?

We have been given M(x)=[tex]4x^{2} -3x[/tex] and N(x)=[tex]-5x^{3}-6x^{2} -3[/tex] and we have to find the difference between M(x) and N(x) and to solve them we have to open the brackets and add the coefficients of variable.

M(x)-N(x)=[tex]4x^{2} -3x-(-5x^{3}-6x^{2} -3)[/tex]

First we have to open the brackets.

=[tex]4x^{2} -3x+5x^{3}+6x^{2} +3[/tex]

Now we have to add the coefficients of same variable having same powers.

[tex]=5x^{3}+10x^{2} -3x+3[/tex]

Hence if M(x)=[tex]4x^{2} -3x[/tex] , N(x)=[tex]-5x^{3}-6x^{2} -3[/tex] then difference between {M(x)-N(x)} =[tex]5x^{3}+10x^{2} -3x+3[/tex].

Learn more about equations at https://brainly.com/question/2972832

#SPJ2

-7 -5(-4c+4) <-7c+8-3

Answers

Answer:

c  <  32/27

Step-by-step explanation:

−7−5(−4c+4)<−7c+8−3

Step 1: Simplify both sides of the inequality.

20c−27<−7c+5

Step 2: Add 7c to both sides.

20c−27+7c<−7c+5+7c

27c−27<5

Step 3: Add 27 to both sides.

27c−27+27<5+27

27c<32

Step 4: Divide both sides by 27.

27c/27  <  32/27

c <  32/27

the answer is c < 32/27

Which equations and/or functions represent the graphed line? Select three options.
4
.
..6
Х
1

Answers

Answer:

f(x) = 0.5x + 2

f(x) = 1/2x + 2

y - 3 = 1/2(x - 2)

y - 1 = 0.5(x + 2)

Step-by-step explanation:

In the figure attached, the graphed line is shown.

The missing options are:

f(x) = 0.2x - 4

f(x) = 0.5x + 2  

f(x) = 1/2x + 2

y – 3 = 1/2(x – 2)

y – 1 = 0.5(x + 2)

From the picture, we can see that points (0, 2) and (2, 3) are on the line. Then, the slope of the line is:

m = (3 - 2)/(2 - 0) = 1/2 = 0.5

The y-intercept is (0, 2), or b = 2

Therefore, in the slope y-intercept form, the equation is:

f(x) = mx + b

f(x) = 1/2x + 2 = 0.5x + 2

In the point-slope form, the equations is:

y - y1 = m(x - x1)

y - 3 = 1/2(x - 2)

Using point (-2, 1), in the point-slope form, the equation is:

y - y1 = m(x - x1)

y - 1 = 0.5(x + 2)

Answer:

bde

Step-by-step explanation:

Board A: 3ft 9in Board B: 4ft 3in Board C: 2ft 6in
If he glues boards A and C together end to end, how long will the new board be?

Answers

The board will be 75 inches long

Consider the statement P: x + 9 = 10. Which of the following is an equivalent statement? Question 13 options: A) R: x > 0 B) Q: x – 9 = 10 C) S: x = 1 D) ∼P: x + 9 ≠ 10

Answers

Answer:

C) S: x = 1

Step-by-step explanation:

Answer:

C) S: x = 1

explination

yall i really need help on this one :/ Mrs. Berry asked her real-estate agent, "What is the median value of homes in this neighborhood?"
Select from the drop-down menus to correctly complete the statement.

Mrs. Berry's question is a statistical question because there ____ variability in the values, and because data _____ to be collected to answer it.
for the first blank the options are is and is no. for the second blank the options are has and do not have.

Answers

non stat,does not varyStep-by-step explanation:


Definition of Complementary Angles

Answers

either of 2 angles whos sum is 90°

Answer:

Add to 90 degrees

Step-by-step explanation:

Complementary angles are angles that add up to 90 degrees or a right angle. For example,

52°+38°=90°

Therefore, 52 and 38 are complementary angles.

the cougars baseball team played 16 games last year and won 12 of them. this year they are scheduled to play 28 games. if the end up with the same winning percentage, how many games will they win

Answers

Answer:

They won 21 games

Step-by-step explanation:

First take the 12 out of 16 games they have won and make it a fraction, 12/16. Then divide 12 by 16, 12 ÷ 16 and then you get, 0.75 then as always multiply it by 100 and get 75 (you can also just move the decimal over 2 places) Then after that you smack that percentage sign on 75 and get %75. So now we know that the Cougars baseball team won %75 of there games. Now take 28 and put it as a fraction, but! You need two numbers for it to be a proper fraction to we will replace the numerator with W to represent how many games they won, so now you should have something like this W/28, so now we find the value of W and by doing that just guess and check (it cant be over 28 so it must be under 28) and when your looking for your number do W ÷ 28 and try to get 0.75 and when you get that (that being 21 ÷ 28 = 0.75 ) you now know that 21 is your answer.

Hope this helps but im in 6th so..... dont quote me on this some of the explanation might not be clear to you but it is to me hope you can figure it out

Find the median of both you and your friend’s basketball scores. Who’s score is greater? By how much?

Answers

Answer:

Find the number in the middle

First list the number from smallest to greatest

6,8,10,10,12,14,14,16,16,18,18,20,20,22

Find which one is in the middle

16 and 16 are the two middle numbers. There is two middle numbers because there is an even amount of numbers

If there are two middle numbers find the average

Do (X+Y) / 2

16+16 = 32

32 / 2 = 16

16 is the median for your score

Do the same to your friend's score

2,8,8,10,10,10,12,12,12,12,14,14,14,16,16

12 is the middle because there is 7 numbers to the left and right so 12 is the median.

Your score is greater by 4 because 16 - 12 = 4.

Hope this helps

Step-by-step explanation:

(4a^4)^2 write without exponents

Answers

(4 x a x a x a x a) x (4 x a x a x a x a)

Answer:

16 a^8 = 16aaaaaaaa

Step-by-step explanation:

(4a^4)^2

We know (xy)^z = x^z y^z

4^2 a^4^2

16 a^4^2

We know a^b^c = a^(b*c)

16 a^(4*2)

16 a^8

Polygons QRST and Q’R’S’T’ are shown on the coordinate grid: A coordinate plane with two polygons is shown. Polygon QRST has vertices Q at 3 comma negative 5, R at 2 comma negative 1, S at 5 comma 0, and T at 5 comma negative 4. Polygon Q prime R prime S prime T prime has vertices at Q prime negative 5 comma negative 4, R prime at negative 1 comma negative 3, S prime at 0 comma negative 6, and T prime at negative 4 comma negative 6. What set of transformations is performed on QRST to form Q’R’S’T’? A 180-degree clockwise rotation about the origin followed by a translation 1 unit to the right A 180-degree clockwise rotation about the origin followed by a translation 1 unit to the left A translation 1 unit to the left followed by a 270-degree counterclockwise rotation about the origin A translation 1 unit to the right followed by a 270-degree counterclockwise rotation about the origin

Answers

Answer:

the answer is D

Step-by-step explanation:

If you take wha it says and plug it in, it makes the most sense

Answer:

d

Step-by-step explanation:

Casey has a litter of 5 puppies and 3 of them are gray. At this rate, how many puppies would be gray in a litter of 30 puppies?

Answers

Answer:

18 gray puppies

Step-by-step explanation:

Think of this as finding equivalent fractions/ratios. 3/5 puppies are gray. what over 30 puppies would be gray? to find 3/5=?/30, you multiply the numerator and denominator by 6. 3 times 6 is 18.

If m∠9 ≅ 95°, find m∠14. answers: Question 5 options: 85° 95° 90° 88°

Answers

Answer:

Option (1). 85°

Step-by-step explanation:

From the figure attached,

'l' and 'm' are the parallel lines and line 'n' is a transverse.

Since, m∠9 ≅ m∠13 [These angles are the corresponding angles]

Therefore, m∠13 = 95°

Since, m∠13 + m∠14 = 180° [Supplementary angles]

Therefore, 95° + m∠14 = 180°

m∠14 = 180° - 95°

m∠14 = 85°

Therefore, 85° will be the measure of angle 14.

Option (1) will be the answer.

Question #19- Use the answer bank below to fill in missing parts of the proof.
Given: PQPR
Prove: QR
P
R

Answers

Answer:

∠MPQ ≅ ∠MPR: Reason; Corresponding parts of congruent triangles are congruent (CPCTC)

∠PQR ≅∠PRQ: Reason; CPCTC

Step-by-step explanation:

[tex]\overline{PQ}\cong \overline{PR}[/tex]: Reason; Given

Draw [tex]\overline{PM}[/tex] so that M is the midpoint of [tex]\overline{QR}[/tex]: Reason; Two points determine a line

[tex]\overline{QM}\cong \overline{RM}[/tex]: Reason; Definition of midpoint

[tex]\overline{PM}\cong \overline{PM}[/tex]: Reason; Reflexive property

ΔPQM ≅ ΔPRM: Reason; Side Side Side (SSS) rule for triangle congruency

∠MPQ ≅ ∠MPR: Corresponding parts of congruent triangles are congruent CPCTC

∠PQR ≅∠PRQ: CPCTC

Using the following image, apply what you have learned about linear pairs and solve for

Answers

It would be 4x+76 = 180 in that order in the boxes. You would add 1 to 75 which got 76. Then you would subtract 76 from 180 and get 104 and then divide that by 4 and you would get 26. So x is 26

The rectangular prisms below have the same size and shape.

What is the combined volume, in cubic inches, of the prisms?

A: 8

B: 16

C: 32

D:64

Answers

A is the answer and all u have to do is nothing
it’s A hoped I helped

Write the slope of DB

Answers

the slope is c/b-a i’m pretty sure

Answer: [tex]\frac{c - 0}{b-a}[/tex]

Other Questions
11. que ellos(recibir).a.Recibenb.RecibisC.recibimos12. Ellos(Ver) la televisin tambin.a.Venb. VeisC.vemos13. Y t (recibir) muchos correos electrnica.Recibeb. Recibesc. recibo- Y _(ver) t la televisin?a. Veb. VesC.veo Brockman Corporation's earnings per share were $3.50 last year, and its growth rate during the prior 5 years was 9.0% per year. If that growth rate were maintained, how many years would it take for Brockman's EPS to triple Which factors led to the economic development of the middle colonies the benefits and problems of capitalism and socialism, and evaluate which system provides the greater overall benefit to society. Pls pls pretty pls do any of these whatever you know pls A(n)_____ is a drawing that comes with a message; it tries to get you to think about a topic or an issue.A. advertisementB. Web siteC. cartoonD. PowerPoint presentation What can characterization reveal to the reader? A. The fears and motivations of the characters B. The central conflict the characters will have to struggle against C. The events that will build to the climax and conclusion of the story D. When and where the story takes place Find the circumference of the circle below. (Use 3.14 for Pi) Suppose that examination of a pro forma reveals that the fifth-year net operating income (NOI) for an income-producing property that you are analyzing is $913,058 (you can assume that this cash flow occurs at the end of the year). If you estimate the projected rental growth rate for the property to be 3% per year, determine the projected sale price of the property at the end of year 5 if the going-out capitalization rate is 8%. Andrew is buying a dirt bike for $900. He will make $100 payments eachmonth until the bike is paid in full. This is an example of what type of loan? Jerusalem is home to the Al-Aqua Mosque and ? Naz and Lara are going to the cinema. The film starts at 15.10 They want to arrive 20 minutes early. It takes them 35 minutes to walk to the cinema. What time should they leave home? why would the symbol of a scale best represent the congress of viennas accomplishments Somebody help ASAP please please please. I basically forgot all the steps to do this. Solve the system of equations.x + 3y = 12x + 2y = 6(4, 1)(2, 1)(4, 1)(5, 2) During a typical Pennsylvania winter, I80 averages 1.6 potholes per 10miles. A certain county is responsible for repairing potholes in a 30-mile stretch of the interstate. LetXdenote the number of potholes thecounty will have to repair at the end of next winter.(a) The distribution of the random variable X is (choose one)(i) binomial (ii) hypergeometric (iii) negative binomial (iv) Poisson.(b) Give the expected value and variance of X.(c)The cost of repairing a pothole is $5000. If Y denotes the countys pothole repair expense for next winter, find the mean value and variance Y? In the diagram below, DE is parallel to XY. What is the value of x?.X105A. 75B. 125C. 105D. 95 If a rectangle has a width of 7 centimeters less than its length, and its area is 330 square centimeters. What are its length and width What is "In an isolated system two cars, each with a mass of 2,000 kg, collide. Car 1 is initially at rest , while car 2 was moving at 20 m/s. what is their combined momentum after the collision?" A negative sanction rewards a particular kind of behavior. Please select the best answer from the choices provided T F PLZZ HELP :)